Você está na página 1de 5

Teorema Chins dos Restos e

Samuel Barbosa

22 de maro de 2006 c
Teorema 1. (Bzout) Sejam a e b inteiros no nulos e d seu mdc. Ento existem inteiros x e y tais que e a a d = ax + by. Se a e b so positivos podemos escolher x > 0 e y < 0, ou vice-versa. a Prova. Seja P = {ax+by|ax+by > 0 e x, y Z}. O conjunto P no vazio pois 0 < a2 +b2 = aa+b e a b P . Seja f o menor elemento de P . Claramente d = mdc(a, b)| f Como f, d > 0, para mostrarmos que d = f basta que f | d. Seja a = qf + r, com q Z e 0 r < f . Assim 0 r = a(1 qx) + b(qy) Z. Como r < f r = 0. Analogamente f | b. Ento f | mdc(a, b) = d. A outra parte deixada para o leitor. a e Teorema 2. Se mdc(a, m) = 1, ento existe um inteiro x tal que ax 1 (mod m). Quaisquer dois tais a x so congruentes (mod m) e se mdc(a, m) > 1 no existe soluo. a a ca Prova. Pelo teorema de Bzout, se mdc(a, m) = 1 existem x e y tais que ax + my = 1. mas isto e signica que ax 1 (mod m). Reciprocamente, se ax 1 (mod m) existe um y tal que ax + my = 1 mdc(a, m) = 1. Se ax1 1 ax2 (mod m) a(x1 x2 ) 0 (mod m), mas mdc(a, m) = 1 m| (x1 x2 ) x1 x2 (mod m) Tal inteiro x chamado de inverso de a mdulo m. Acabamos de mostrar que se mdc(a, m) = 1, o e o inverso de a existe unico mdulo m. Dizemos que os inteiros a1 , a2 , . . . , am so primos entre si, dois a e o a dois, se mdc(ai , aj ) = 1 quando i = j. Vejamos nosso resultado principal: Teorema 3. (Teorema Chins dos Restos) Sejam m1 , m2 , . . . , mr , r inteiros positivos que so primos e a entre si, dois a dois, e sejam a1 , a2 , . . . , ar , r inteiros quaisquer. Ento, o sistema de conguncias: a e x a1 x a2 (mod m1 ) (mod m2 ) . . . x ar (mod mr ) admite uma soluo x. Alm disso, as solues so unicas mdulo m = m1 m2 . . . mr . ca e co a o m m um inteiro e mdc e , mj = 1. Ento pelo a mj mj m m bj 1 (mod mj ). Claramente bj 0 teorema 2, para cada j, existe um inteiro bj tal que mj mj (mod mi ) para i = j. Denamos r m x0 = bj aj mj j=1 Prova. Escrevendo m = m1 m2 . . . mr , vemos que m bj aj ai (mod mi ). Ento x0 uma soluo do nosso sistema. a e ca mi Se x0 e x1 so solues do nosso sistema ento: x0 x1 (mod mi ) para cada i. Como mdc(mi , mj ) = a co a 1 se i = j ento x0 x1 (mod m). a Consideremos x0 mdulo mi : x0 o 1

Exemplo 1. Encontre o menor inteiro positivo x tal que x 5 (mod 7), x 7 (mod 11) e x 3 (mod 13). Usando o teorema anterior com m1 = 5, m2 = 7, m3 = 11, a1 = 5, a2 = 7 e a3 = 3 podemos achar x 887( mod 1001 = 7.11.13). Como a soluo unica mdulo m, isto signica que, dentre os nmeros ca e o u 1, 2, , 1001 a menor soluo positiva 887. ca e Exerc cio 1. (Estnia 2000) Determine todos os restos poss o veis da diviso do quadrado de um nmero a u primo com 120 por 120. Aconselhamos que o leitor faa alguns exemplos numricos at se acostumar com o algoritmo usado c e e para encontrar x0 . Provamos no teorema passado que todas as solues daquele sistema de congruncias co e so os termos de uma P.A de razo m. Em geral, usamos o teorema 3 apenas para garantir que um a a sistema de congruncias admite uma soluo. Os prximos exemplos devem deixar isto mais claro. e ca o Exemplo 2. Para cada nmero natural n, existe uma sequncia arbitrariamente longa de nmeros natuu e u rais consecutivos, cada um deles sendo divis por uma s-sima potncia de um nmero natural maior vel e e u que 1. Prova. Dado m N considere o cunjunto {p1 , p2 , . . . , pm } de primos distintos. Como mdc(ps , ps ) = 1, i j ento pelo teorema 3, existe x tal que x i (mod ps ) para i = 1, 2, . . . m. Cada um dos nmeros do a u i conjunto {x + 1, x + 2, . . . , x + m} divis por um nmero da forma ps . e vel u i Exemplo 3. (USAMO 1986) (a) Existem 14 inteiros positivos consecutivos tais que, cada um divis por um ou mais primos p do e vel intervalo 2 p 11? (b) Existem 21 inteiros positivos consecutivos tais que, cada um divis por um ou mais primos p do e vel intervalo 2 p 13? Soluo. (a) No. Suponha que existam tais inteiros. Da nossa lista de 14 inteiros consecutivos, 7 so ca a a nmeros pares. Vamos observar os u mpares: a, a + 2, a + 4, a + 6, a + 8, a + 10 e a + 12. Podemos ter no mximo trs deles divis a e veis por 3, dois por 5, um por 7 e um por 11. Veja que 3 + 2 + 1 + 1 = 7. Pelo Princ pio da Casa dos Pombos, cada um desses mpares divis e vel por exatamente um primo do conjunto {3, 5, 7, 11}. veja que os mltiplos de 3 s podem ser {a, a + 6, a + 12}. Dois dos nmeros u o u restantes (a + 2, a + 4, a + 8, e a + 10) so divis a veis por 5. Mas isto imposs e vel. (b) Sim. Como os nmeros {210, 11, 13} so primos entre si, dois a dois, pelo teorema 3 existe um inteiro positivo n > 10 u a tal que: n 0( mod 210 = 2 3 5 7) n 1( mod 11) n 1( mod 13) Veja que o conjunto {n 10, n 9, . . . , n + 9, n + 10} satisfaz as condies do item (b). co Exemplo 4. (Olimp ada de So Petesburgo 1990) Dado um polinmio F (x) com coecientes inteiros, a o tal que, para cada inteiro n, o valor de F (n) divis por pelo menos umm dos inteiros a1 , a2 , , am . e vel Prove que podemos encontrar um ndice k tal que F (n) divis por ak para cada inteiro positivo n. e vel Soluo. Suponha que no exista tal ca a ndice. Para cada ndice k (k = 1, 2, . . . , m) existe um inteiro xk , tal que , F (xk ) no divis por ak . Assim, existem nmeros dk = pk onde os pk so primos, tais que, a e vel u a k dk divide ak mas no dividem F (xk ). Se exitem potncias do mesmo primo entre esses nmeros, podemos a e u apagar aquelas deixando apenas uma que tem expoente m nimo. Caso F (x) no seja divis por uma a vel 2

potncia apagada, no ser pela potncia que tem expoente m e a a e nimo. Essa delees garatem que nossa nova co coleo d1 , d2 , . . . , dj de potncias de primos contm apenas inteiros primos entre si, dois a dois. Pelo ca e e teorema 3, exite um inteiro N , tal que, N xk ( mod dk ) para k {1, 2, . . . , j}. Suponhamos que dk | F (N ). Sabemos que x y| F (x) F (y) N xk | F (N ) F (xk ). Mas dk | N xk e dk | F (N ) dk | F (xk ). Uma contradio! Logo F (N ) no divispivel por nenhum dk , mas isto contradiz a hiptese sobre os ai . ca a e o Teorema 4. (Paul Erds) Existe uma progresso aritmtica innita de nmeros o a e u mpares, nenhum deles da forma 2k + p, onde k Z+ , e p um primo. e Lema. Todo nmero natural satisfaz pelo menos uma das seguintes congruncias: u e (1) k 0( mod 2) (4) k 3( mod 8) (2) k 0( mod 3) (5) k 7( mod 12) (3) k 1( mod 4) (6) k 23( mod 24)

Prova. Se um nmero k no satisfaz (1) ou (2), ento no divis por 2 ou 3.Ento deve ser da forma u a a a e vel a 24t + r onde t um inteiro e r m dos nmeros 1, 5, 7, 11, 13, 17, 19, 23. Uma vericao direta mostra e e u ca que k tem que satisfazer as congruncias (3), (3), (5), (4), (3), (3), (4), (6), respectivamente. e Corolrio. Se k um inteiro no-negativo, ento pelo menos uma das seguintes congruncias satisfeita: a e a a e e (7) 2k 1( mod 3) (10) 2k 23 ( mod 17) (8) 2k 1( mod 7) (11) 2k 27 ( mod 13) (9) 2k 2( mod 5) (12) 2k 223 ( mod 241)

Prova. Basta vericarmos que 22 1( mod 3) , 23 1( mod 7), 24 1( mod 5) , 28 1( mod 17), 212 1( mod 13), 212 1( mod 241), ento 224 1( mod 241). Devido ` estas, as congruncias a a e (1), (2), (3), (4), (5), (6) implicam em (7), (8), (9), (10), (11), (12), respectivamente. Prova do Teorema. Em virtude do Teorema Chins dos Restos, existe um natural a que satisfaz as e congruncias: e a 1( mod 2 = m1 ) a 2( mod 5 = m4 ) a 223 ( mod 241 = m7 ) a 1( mod 3 = m2 ) a 23 ( mod 17 = m5 ) a 3( mod 31 = m8 ) a 1( mod 7 = m3 ) a 27 ( mod 13 = m6 )

Alm disso, existem innitas progresses aritmticas de as que satisfazem essas congruncias. Todas e o e e essas progresses tem razo mltipla de m , onde m = m1 m2 . . . m8 , como no enunciado do teorema 3. o a u Claramente, os termos dessas progresses so o a mpares. Se a qualquer termo de uma dessas progresses, o e o corolrio do lema diz que a2k divis por pelo menos um dos primos 3, 7, 5, 17, 13, 241. Por outro lado, a e vel a 3( mod 31) e para qualquer k Z+ o nmero 2k congruente a um dos nmeros 1, 2, 4, 8( mod 31). u e u Consequentemente, a2k congruente a um dos nmeros 2, 1, 9, 5, 13( mod 31). Mas nenhum desses e u nmeros congruente mod 31 a qualquer um dos nmeros 3, 7, 5, 17, 13, 241. Ento o nmero a 2k no u e u a u a pode ser 3, 7, 5, 17, 13 ou 241, por outro lado, divis por pelo menos um deles. Ento um nmero e vel a e u composto. Assim no existe um primo p tal que a = 2k + p. a Corolrio. (Sierpi ski) Existem innitos nmeros naturais n tais que, cada um dos nmeros n2k + 1, a n u u k Z+ , composto. e Prova. A prova do teorema anterior, mostra que existem innitos naturais n tais que para qualquer inteiro no-negativo k, o nmero n 2k (ou melhor dizendo 2k + n) divis por pelo menos um dos a u e vel nmeros 3, 7, 5, 17, 13, 241. Seja P o produto de todos esses primos. Em virtude do provado acima, o u nmero n + 2k[(P )1] admite um divisor primo p | P . Mas 2k(P ) 1( mod P ), como n + 2k[(P )1] 0 u ( mod p) n2k + 1 0( mod p). Claramente este numero composto se n > 241 p. e O prximo exemplo uma generalizao de um problema do Banco da IMO de 1992. o e ca

Exemplo 5. Prove que dado n N existe um conjunto de n elementos A N tal que para todo B A, B = , x um potncia no trivial (isto , um nmero da forma mk , onde m e k so inteiros maiores e e a e u a
xB

ou iguais a 2). Soluo. A = {4} e A = {9, 16} so solues para n = 1 e n = 2 respectivamente. Estes sero nossos ca a co a caso iniciais de induo. Suponha que A = {x1 , x2 , , xn } um conjunto com n elementos e para todo ca e B A, B = , x = mkB . Vamos mostrar que existe c N tal que A = {cx1 , cx2 , , cxn , c} B
xB

satisfaz o enunciado. Seja l = mmc{kB , B A, B = }. Para cada B A, B = associemos um primo pB > l, de forma que B1 = B2 pB1 = pB2 , e associemos um natural r com rB 0(mod pX ), X = B, lrB + 1 0(mod pB ) (tal rB existe pelo Teorema Chins dos Restos). Dena c = e (1 + mkB )lrB B
BA B=

Como c uma potncia l-sima, c um potncia kB -sima para todo B A, B = , portanto, para e e e e e e B {cx1 , cx2 , , cxn }, B = teremos B = {cx|x B} para algum B A, B = . Logo x ser a
xB

uma potncia kB -sima. Alm disso, e e e x=


xB {c}
XA X=,B

(1 + mkB )lrX (1 + mkB )lrb +1 X b

uma pB -sima potncia. e e e

Problemas
Problema 1. Existem n inteiros consecutivos tal que cada um contm um fator primo repetido k vezes? e Problema 2. Um ponto (x, y) Z 2 legal se mdc(x, y) = 1. Prove ou disprove: Dado um inteiro positivo e n, existe um ponto (a, b) Z 2 cuja distncia a todo ponto legal e pelo menos n? a Problema 3. Sejam mo , m1 , ..., mr inteiros positivos que so primos entre si, dois a dois. Mostre que a existem r + 1 inteiros consecutivos s, s + 1, ..., s + r tal que mi divide s + i para i = 0, 1, ..., r. Problema 4. Seja P (X) um polinmio com coecientes inteiros e k um inteior qualquer. Prove que o e existe um inteiro m tal que P (m) tem pelo menos k fatores primos distintos. Problema 5. (Koria 1999) Encontre todos os inteiros n tais que 2n 1 um mltiplo de 3 e e e u um divisor de 4m2 + 1 para algum inteiro m. 2n 1 e 3

Problema 6. (Romnia 1995) Seja f : N {0, 1} N denida por f (n) = mmc[1, 2, ..., n]. Prove que e para todo n 2, existem n nmeros consecutivos para os quais f constante. u e Problema 7. (Olimp ada Nrdica 1998) o (a) Para quais inteiros positivos n existe um sequncia x1 , x2 , . . . , xn contendo cada um dos inteiros e 1, 2, . . . , n exatamente uma vez, e tal que k divide x1 + x2 + + xk para k = 1, 2, , n? (b) Existe uma sequncia innita x1 , x2 , . . . contendo todo inteiro positivo exatamente uma vez, e tal que e para cada inteiro positivo k, k divide x1 + x2 + + xk ? Problema 8. Seja n um nmero natural arbitrrio. Prove que existe um par de naturais (a, b) tais que u a mdc(a + r, b + s) > 1 r, s = 1, 2, . . . , n.

Problema 9. (OBM 2005) Dados os inteiros positivos a, c e o inteiro b, prove que existe um inteiro positivo x tal que ax + x b (mod c). Problema 10. (Cone Sul 2003) Demonstrar que existe uma sequncia de inteiros positivos x1 , x2 , . . . que e satisfaz as duas condies seguintes: co (a) contm exatamente uma vez cada um dos inteiros positivos, e (b) a soma parcial x1 + x2 + . . . xn divis por nn . e vel Problema 11. (Repblica Tcheca e Eslovaca 1997) Mosrte que existe uma sequncia crescente {an } u e n=1 de nmeros naturais tais que para k 0 , a sequncia {an + k} contm um nmero nito de primos. u e e u Problema 12. Considere o inteiro c 1 e a sequncia denida por a1 = c e ai+1 = cai . Mostre que e esta sequncia se torna eventualmente constante quando reduzimos mdulo n para algum inteiro positivo e o n (isto signica que am aj (mod n) se m j). Problema 13. (Putnam 1994) Pra qualquer intiro positivo a, seja na = 101a 100 2a . Mostre que para 0 a, b, c, d, 99, na + nb nc + nd (mod 10100) implica {a, b} = {c, d}. Problema 14. Seja an a sequncia denida por e an = 1999 se n = 1, an1 + p(n) se n > 1

onde p(n) o menor divisor primo de n. Mostre que an possui innitos mlriplos de 7. e u Problema 15. Considere a sequncia de inteiros positivos {an }, n = 1, 2, 3, . . . satisfazendo a condio e ca 0 < an+1 an 2001 para todo n = 1, 2, 3, . . .. Prove que existe um nmero innito de pares de inteiros positivos (p, q) tais que u p < q e ap um divisor de aq . e Problema 16. O conjunto S = {1/r : 1, 2, 3, . . .} contm progresses aritmticas de vrios tamanhos. e o e a Por exemplo, {1/20; 1/8; 1/5} uma de tais progresses, de tamanho 3 (e razo 3/40). Mais ainda, essa e o a uma progresso maximal em S de tamanho 3 pois ela no pode ser estendida ` esquerda ou ` direita e a a a a (1/40 e 11/40) no so elementos de S. Mostre que existe uma progresso maximal em S de tamanho a a a m para todo m 3.

Você também pode gostar